Traci needs to change $300 US Dollars into Euros. the exchange rate is 1 euro for 1.25 dollars. how many euros does Traci get for her dollars?

Answers

Answer 1

Let's begin by identifying key information given to us:

Amount = $300

Amount (after changing) = ?

Exchange rate: €1 = $1.25

We will calculate how much Euros Traci gets for her dollar using simple proportion:

[tex]\begin{gathered} x=\text{\$}300 \\ \euro1=\text{\$}1.25 \\ \text{Cross multiply, we have:} \\ 1.25(x)=300\cdot1 \\ x=\frac{300}{1.25}=240 \\ x=\euro240 \end{gathered}[/tex]

Therefore, Traci gets €240 for her dollars

Answer 2
300 is the correct answer

Related Questions

Side PV measures 35 meters. Find the value of VT rounded to thenearest tenth if necessary.

Answers

By definition,

sin(angle) = opposite/hypotenuse

From the picture,

sin(60°) = VT/PV

√3/2 = VT/35

√3/2*35 = VT

30.3 meters = VT

question will be in picture

Answers

Consider the guven system of equations,

[tex]\begin{gathered} x-y=-1 \\ 2x+y=4 \end{gathered}[/tex]

It is asked to find the correct graph which represent the solution of this system.

Logic: Find the solution and see which graph gives the same solution.

Add the equations,

[tex]\begin{gathered} (x-y)+(2x+y)=-1+4 \\ 3x+3 \\ x=1 \end{gathered}[/tex]

Substitute this value in the first equation,

[tex]\begin{gathered} 1-y=-1 \\ y=1+1 \\ y=2 \end{gathered}[/tex]

Thus, the given system has a unique solution (1,2).

Now, observe that only the graph given in option (c) shows the line intersecting at point (1,2).

Therefore, option (c) is the correct choice.

The note A has a frequency of 1,760 hertz. The note D has a period of 1,175 hertz. Find theratio of A to D to two decimal places. Express the answer in integer ratio form.

Answers

Ratios

Note A has a frequency of

fa=1,760 Hz

Note D has a period of 1,175 hertz

(the previous data should be frequency, not period)

We are required to find the ratio of A to D. Let's call it r:

[tex]r=\frac{1,760^{\prime}}{1,175}[/tex]

Dividing: r = 1.4978. Rounding to two decimal places:

r = 1.50

Now to express the answer in integer ratio form, we need to simplify the fraction.

First, we divide by 5 up and down:

[tex]r=\frac{352^{\prime}}{235}[/tex]

There are no more common divisors for both numbers, thus the integer ratio form is r = 352/235

A bag holds 5.8lb of corn. How many pounds do 2.5 bags hold

Answers

ANSWER:

14.5 pounds

STEP-BY-STEP EXPLANATION:

We know the capacity of each bag, therefore, we can calculate the capacity of 2.5 bags, multiplying by the capacity of one, like this:

[tex]\begin{gathered} c=5.8\cdot2.5 \\ \\ c=14.5\text{ lb} \end{gathered}[/tex]

Which means 2.5 bags contain a total of 14.5 lbs.

Solve for t and graph the solution.|t+ 4| < 2

Answers

The solution to the inequality is (-6, -2). The graph of the solution is attached below.

We are given an inequality. The inequality consists of a variable "t" and it also uses the modulus function. A function connects an input with an output. It is analogous to a machine with an input and an output. And the outcome is somehow connected to the input. The inequality is |t + 4| < 2. We can solve the inequality by expanding the modulus operator. The inequality becomes -2 < (t + 4) < 2. Now we will subtract 4 from all the sides in the inequality. The inequality is solved as -6 < t < -2. The interval notation for the solution is (-6, -2). The graph is attached below.

To learn more about modulus, visit :

https://brainly.com/question/13103168

#SPJ9

solve for x[tex] \frac{x + 3}{2} + \frac{2x}{7} = 7[/tex]just need some help with this

Answers

Here, we are to calculate for the value of x.

We start by finding the Lowest common multiples of both fractions. The lowest common multiple of both fractions is 14

Now, we can multiply each of the terms in the question by 14. Thus, we have;

14(x + 3)/2 + 14(2x/7) = (7 * 14)

7(x + 3) + 2(2x) = 98

Opening the brackets, we have;

7x + 21 + 4x = 98

Collect like terms;

7x + 4x = 98 - 21

11x = 77

x = 77/11

x = 7

selecting among the numbers 1 through 8 and repeating none of them, fill in the boxes below to make the sum as close as possible to one but not equal to one

Answers

We need ti fill in the boxes below, selecting among the numbers 1 through 8 and repeating none of them to make the sum as close as possible to 1 but not equal to 1.

First fraction could be 1/2

Now, the second fraction should be closer to 1/2 to make the sum as close as possible to 1, but not equal to 1.

We know that 3/6 = 1/2

So, 3/7 would be closer to 1/2

Therefore, the boxes could be filled as;

[tex]\frac{1}{2}+\frac{3}{7}[/tex]

..

Challenge The value of a baseball player's rookie card began to increase once the player retired. When he retired in 1996 his card was worth $9.17. The value has increased by $1.65 each year since then, Express the relationship relating the value of the card y in dollars and the number of years x the player has been in retirement with an equation. Is the relationship between x and y proportional? What was the value of the card in 2006? Express the relationship with an equation. y=?

Answers

$151.31. Not proportional.

y= 9.17 +1.65x

1) Gathering the data

1996 Retired $ 9.17

9.17(1.65)^x

2) Since the value of that card has been increasing 1.65 yearly, then we can write the following equation:

[tex]y=9.17+1.65x[/tex]

For y= Value, and x =years

Let's find out the value of the card in 2006, i.e. 10 years later. Plug x=10 into the above function:

[tex]\begin{gathered} y=9.17+1.65x \\ y=9.17+1.65(10) \\ y=9.17+16.5 \\ y=151.305\approx y=151.31 \end{gathered}[/tex]

3) Hence, the value of the card in 2006 is $151.31

(rounded off to the nearest hundredth)

And since the initial value is $9.17, then it is not a proportional relationship since an initial value of a proportional is always 0.

Simplify each expression using Product to a Power Property a. (9a)^2 b. ( 3x)^2

Answers

[tex]\begin{gathered} a)81a^2 \\ b)9x^2 \end{gathered}[/tex]

1) Let's simplify those expressions making use of exponents rules:

[tex](9a)^2=81a^2[/tex]

Note that the exponent outside the parenthesis is 'distributed' over each term inside.

2) Similarly for the second expression we can state that:

[tex](3x)^2=9x^2[/tex]

A survey wanted to determine if there was a relationship between the number of joggers who used a local park for exercise and the temperature outside. The data in the table display their findings.Use graphing technology to create a scatter plot of the data.What is the slope of the line of best fit and what does it mean in this context? Is it realistic?

Answers

Step 1

In order to find the slope of the line of the best fit, we must graph the points given.

Step 2

From the image above we can conclude that the equation of the line is;

[tex]y=0.410045x-2.8757[/tex]

The slope is, therefore;

[tex]0.410045[/tex]

The Slope (or Gradient) we call m, represents the change in y-value per unit change in x-value. In the case of this survey, the slope represents the increase in the number of joggers per degree rise in temperature.

From the calculated slope, there are about 0.4 more joggers for every degree rise in temperature. Using whole numbers to represent this, we can multiply the slope by 5:

[tex]0.4\times5=2[/tex]

Therefore, there are 2 more joggers for every 5°F increase in temperature.

I need help with this

Answers

Ok, so

We have the following expression:

We're going to multiply all terms inside the bracket by "-x", and we obtain:

So, the answer is B.

what does ☆ equal if the symbols below represents digits 0-9?

Answers

First, let's start from the sixth equation.

Since the parallelogram divided by the circle is equal the parallelogram, it means the circle has a value of 1.

Then, from the 7th equation, the pentagon minus the arrow is equal the pentagon, it means the arrow is equal 0.

From the 8th equation, the two curved arrows summed are equal 10, so the curved arrow is equal 5.

From the 2nd and 4th equations we can find that the triangle is equal 2.

Using the value of the pentagon from the 4th equation in the 3rd one, we have that 3 * moon is equal parallelogram, so the only option is:

moon = 3, pentagon = 6, parallelogram = 9.

From the 1st equation, the square is equal 8.

Finally, from the 5th equation, the arrow with circle is equal 7.

So we have:

pentagon = 6

parallelogram = 9

moon = 3

triangle = 2

arrow = 0

curved arrow = 5

arrow with circle = 7

circle = 1

square = 8

So the star is the missing algarism: 4

unsure about this one tried and couldn’t seem to figure it out

Answers

Given:

The figure contains the rectangle and half-circle.

The perimeter of the half-circle is,

[tex]\begin{gathered} \text{Half}-\text{circle}=\pi r+d \\ r=\frac{d}{2}=\frac{8}{2}=4 \\ \text{Half}-\text{circle}=\pi(4)+8 \\ =3.14\times4+8 \\ =20.56 \end{gathered}[/tex]

The perimeter of the figure is,

[tex]P=10+8+10+20.56=48.56[/tex]

Answer: P = 48.56 ft.

Question #10: A pizza restaurant sells a personal square pizza (5 inch side) for $4. What should the cost of a large square pizza (12 inch side) be, so that both pizzas cost the same amount per square inch?

Answers

The per square-inch cost of the first kind of pizza is $0.16 to maintain this ratio for 12-inch side pizza the cost must be $23.04.

What are the ratio and proportion?

The ratio is the division of the two numbers.

For example, a/b, where a will be the numerator and b will be the denominator.

Proportion is the relation of a variable with another. It could be direct or inverse.

As per the given,

Cost of first pizza = $4

Area = 5 x 5 = 25 square inch

Per square cost = 4/25 = $0.16

With this ratio,

Cost of 12 inches side = 12 x 12 x 0.16 = $23.04

Hence "The per square-inch cost of the first kind of pizza is $0.16 to maintain this ratio for 12-inch side pizza the cost must be $23.04".

For more information about ratios and proportions,

brainly.com/question/26974513

#SPJ1

what is (x + 5)(2× - 3) = 0 in expanded form??

Answers

(x + 5)(2x -3) = 0

2x² - 3x + 10x - 15 = 0

2x² + 7x - 15 = 0

Answer:

2x² + 7x - 15 = 0

The function h is defined below.xh (x) =2+5x-- 142x- 81Find all values of × that are NOT in the domain of h.If there is more than one value, separate them with commas.

Answers

We want to find the value of x that is not in the domain of the function;

[tex]h(x)=\frac{x^2+5x-14}{x^2-81}[/tex]

The value of x that will make h undefined is the value that makes the denominator zero.

Therefore;

[tex]\begin{gathered} x^2-81=0 \\ x^2=81 \\ x=\pm9_{} \end{gathered}[/tex]

Therefore, the values not in the domain is +9 and -9

Over the summer, Audrey had a job and earned $528. She spent $125.70 on a new bike and $85.09 for some new clothes. How much money does Audrey have left?

Answers

Step 1: Problem

Over the summer, Audrey had a job and earned $528. She spent $125.70 on a new bike and $85.09 for some new clothes. How much money does Audrey have left?​

Step 2: Concept

Word problem on subtraction

Step 3: Method

Audrey total earning = $528

Money spent on new bike = $125.7

Money spent on new clothes = $85.09

Money left = 528 - 125.7 - 85.09

= $317.21

Step 4: Final answer

Audrey money left = $317.21

Write a recursive sequence that represents the sequence defined by the following explicit formula: an=-405(-1/3)^n+1

Answers

[tex]\begin{gathered} \text{ an = -405(-}\frac{1}{3})^{n\text{ + 1}} \\ \text{ a}_1\text{ = -405(-}\frac{1}{3})^{1\text{ + 1}}\text{ = -405(-}\frac{1}{3})^2\text{ = -405(}\frac{1}{9})\text{ = -45} \end{gathered}[/tex]

ok

[tex]\begin{gathered} \text{ an = -405(-}\frac{1}{3})^{n\text{ + 1 - 1}} \\ \text{ an = -405(-}\frac{1}{3})^n \end{gathered}[/tex]

is this a positive, negative or no correlation? please help

Answers

Answer:

Positive correlation!

Step-by-step explanation:

Write the ratio as a fraction in simplest form without any units.9 gallons to 28 quarts

Answers

Given

The ratio, 9 gallons to 28 quarts.

To find:

The ratio as a fraction in lowest term.

Explanation:

It is given that,

The ratio, 9 gallons to 28 quarts.

That implies,

[tex]\begin{gathered} 9\text{ }gallons\text{ }to\text{ }28\text{ }quarts=\frac{9\text{ }gallons}{28\text{ }quarts} \\ =\frac{9\times4\text{ }quarts}{28\text{ }quarts} \\ =\frac{9}{7} \end{gathered}[/tex]

Hence, the simplest form of the ratio is 9/7.

students at a local school were asked. “about how many hours do you spend on homework each week “?The table shows the results of the survey . classify the statement below as true or false .more students study for 5 to 6 hours than for 1 to 2 hours

Answers

From the table:

The number of students that study for 5 to 6 hours = 109

The number of students that study for 1 to 2 hours = 142

Since 109 is less than 142:

The statement is False because 109 students study for 5 to 6 hours and 142 students study for 1 to 2 hours.

if y=3x+4 and y=2x+8, find the value of both x and y.

Answers

[tex]\begin{cases}y=3x+4 \\ y=2x+8\end{cases}\rightarrow3x+4=2x+8\rightarrow3x-2x=4\rightarrow x=4[/tex]

having that x=4, we get that

[tex]y=2\cdot4+8=16[/tex]

What is the perimeter of the figure2x + 3 inchesX- 6 inches

Answers

Perimeter of the figure = 6x - 6

Explanation:

Perimeter = 2(length + width)

length = 2x + 3 inches

width = x- 6 inches

Perimeter = 2(2x + 3 + x- 6)

collect like terms in the bracket:

= 2(2x +x + 3 - 6)

= 2(3x - 3)

Expand the bracket:

Perimeter = 2 × 3x + 2× -3

Perimeter of the figure = 6x - 6

In which quadrant, or on which axis, does the terminal side of angle (- 100straight pi) lie?

Answers

The given angle is

[tex]\theta=-\frac{100}{\pi}[/tex]

First, we have to transform this angle into degrees. We know pi=180°, so

[tex]\theta=-\frac{100}{180}=-\frac{50}{90}=-\frac{5}{9}=-0.56[/tex]The given angle is placed on the first quadrant.

Assuming that the angle is starting on the positive x-axis.

J is the midpoint of HK, H has coordinates (5,-3), and J has coordinates (7,3). Find the coordinates of K.The coordinates of K are

Answers

Answer:

The coordinates of K is;

[tex](9,9)[/tex]

Explanation:

We want to find the coordinates of point K.

Given that J is the midpoint of HK and;

H has coordinates (5,-3)

J has coordinates (7,3).

The formula for calculating midpoint is;

[tex]\begin{gathered} x=\frac{x_1+x_2}{2} \\ y=\frac{y_1+y_2}{2} \end{gathered}[/tex]

where x1 and x2 are the x coordinates of the endpoints and y1 and y2 are the y coordinates of the endpoints.

To get the coordinates of one of the endpoints, we have;

[tex]\begin{gathered} x_2=2x-x_1 \\ y_2=2y-y_1 \end{gathered}[/tex]

substituting the given coordinates of the mid point and endpoint;

[tex]\begin{gathered} x_2=2(7)-5 \\ x_2=14-5 \\ x_2=9 \end{gathered}[/tex][tex]\begin{gathered} y_2=2(3)-(-3) \\ y_2=6+3 \\ y_2=9 \end{gathered}[/tex]

Therefore, the coordinates of K is;

[tex](9,9)[/tex]

The total annual sales for Herman's Hardware Store was $1,246,135 and the total accounts receivable was $41,728. What was the average collection period to the nearestwhole day?12 days14 days18 days24 daysNone of these choices are correct.

Answers

The average collection Period formula is

[tex]\text{Average collection period=}\frac{Account\text{ receivable}}{\frac{Annual\text{ sales}}{365}}[/tex][tex]\begin{gathered} \text{Annual sales=\$1,246,135} \\ \text{Account receivable =\$41,728} \end{gathered}[/tex]

Substitute the values above in the average collection period

[tex]\begin{gathered} \text{Average collection period =}\frac{41728}{\frac{1246135}{365}} \\ =\frac{41728}{3414.06} \\ =12.222 \\ \approx12days \end{gathered}[/tex]

Hence the average collection period to the nearest whole day is 12 days

The lines represented by the equations y = 1/2x-8 and 2y+4x=10

Answers

The first equation is in slope-intercept form but the second equation is not. Therefore, our first step is to bring 2y+4x=10 into slope-intercept form.

Subtracting 4x from both sides gives

[tex]2y=10-4x[/tex]

finally, dividing both sides by 2 gives

[tex]y=-2x+5[/tex]

Now, that we have the equation in slope-intercept form, let us compare their slopes to see how they relate to each other.

The lines are perpendicular if the slope of one line is negative of the reciprocal of the other.

Now, -2 is negative of the reciprocal of 1/2. Meaning if you take the reciprocal 1/2, then multiply it by -1 you will end up with -2.

Since the slope of a negative reciprocal of each other, the lines are perpendicular.

how do you dilate with a scale factor of -1/2Coordinates (-2,8)

Answers

Multiply the coordinates by the scale factor.

(-2,8) = (-2x-1/2, 8x-1/2) = (1,-4)

A builder is buying boxes of nails. She has $187 to spend and each box of nails costs $4. How many boxes of nails can the builder buy?42503746

Answers

To find How many boxes of nails can the builder buy divide the total she has into the cost of each box:

Then, she can buy 46 boxes of nails

Please help me and explain how to find the result

Answers

A family consumes 1.2kg of rice a day

If the family have 30ky of rice

Let the number of days be x

The minimum number of days required for the amount of rice remaining not to be more than 6kg can be expressed below as

[tex]30-1.2x\le6[/tex]

Solve for x by collecting like terms

[tex]\begin{gathered} 30-1.2x\le6 \\ 30-6\le1.2x \\ 24\le1.2x \\ 1.2x\ge24 \\ \text{Divide both sides by 1.2} \\ \frac{1.2x}{1.2}\ge\frac{24}{1.2} \\ x\ge20\text{ days} \end{gathered}[/tex]

Hence, the minimum number of days required for the amount of rice remaining not to be more than 6kg is 20 days

Other Questions
A ball is sliding from the top to the bottom of a plank without rolling (e.g. imagine the surface is covered in ice, so very slippery). The ball is returned to the top and released again, but this time the ball is rolling (without slipping) down the plank (imagine the ice has melted). Compare the speeds of the ball at the bottom.a.The final speed is the same in both cases.b.The final speed is larger in the second case (with rolling).c.The final speed is larger in the first case (without rolling).d.The final speed is larger in the first case (without rolling) if the the plank is at an angle bigger than 45o and smaller if the angle is less than that. A sample of helium gas initially at 0.90 atm is cooled from 18 degreeC to -48 degreeC at constant volume. What is the final pressure? What is the length of side s of the square shown below?456S90A. 2.B. 6C. 3D. 5.2E. 3.2F. .6 According to the diagram, an 8-foot-tall statue casts a shadow on the ground that is 15 feet in length. Based on this information, which trigonometric ratio has the value 8/15 ?A. cos CB. tan BC. cos BD. tan C A 5.2 105 kg subway train is brought to a stop from a speed of 0.55 m/s in 0.51 m by a large spring bumper at the end of its track. What is the force constant k of the spring in N/m? What is the definition of function?Hos inputs andoutputsInputs haveEvery input hosonly ONE outputxrches andy-wolvesdifferent outputsevery time A Ferris wheel with a 200-foot diameter is spinning at a rate of 10 miles per hour. Find the angular speed of the wheel in radians per minute. the journal entry to record depletion includes a a. credit to accumulated depreciation. b. debit to accumulated depletion. c. debit to depletion expense. d. credit to depletion expense. What is the equation of the line below Could you please check my answer? Use the equations to solve the system of equations:Y=0X=7My answer: (7,0) Evaluate: 4+8/2 x (6 - 3)163325 what is the primary binding mechanism of a band level society? in other words, what usually is most effective in keeping these societies from disintegrating? Find the highest common factor (HCF) of 32, 48 and 72 Six office desks that are 7 1/12 feet long are to be placed together on a wall that is 42 7/12 feet long. Will they fit on the wall? Select the correct choice below and, if necessary, fill in the answer box to complete your choice. O A. Yes, if no more than a total of foot is needed for spacing between desks. (Type an integer or a simplified fraction.) B. No, they do not all fit along the wall. A triangle has angles that are 38 and 47. Find the measure of the third angle. 177 95 133 85 May 10, 12:38:01 AMA survey was given to a random sample of 195 residents of a town todetermine whether they support a new plan to raise taxes in order toincrease education spending. Of those surveyed, 39 respondents saidthey were in favor of the plan. At the 95% confidence level, what is themargin of error for this survey expressed as a proportion to thenearest thousandth? (Do not write +).Submit AnswerAnswer: hi! I have the answers to A. AND B. which are k=1.243% for a and k= 0.20% for bWhere y is the population after t time, A is the initial population and k is the growth constant.Therefore, for each case, we calculate the value of k:(a)t = 4y = 1375000A = 1309000Solving for k:1375000=1309000ek4e4k=137500013090004k=ln(13750001309000)k=ln(13750001309000)4k=0.012290.01231.23%(b)t = 4y = 1386000A = 1375000Solving for k:1386000=1375000ek4e4k=138600013750004k=ln(13860001375000)k=ln(13860001375000)4k=0.001990.0020.20%(c)To compare we calculate the quotient between both periods: is something is greater than 6 , do i include 6 which is a common cause(s) of congenital obstructions in the lower urinary tract? select all that apply. meatal stenosis bladder tumors meningomyelocele spina bifida enlarged prostate 9. You want to be able to withdraw the specified amount periodically from a payout annuity with the given terms. Find how much the account needs to hold to make this possible. Round your answer to the nearest dollar. Regular withdrawal: $4500 Interest rate: 4.5% Frequency quarterly Time: 24 years Account balance: $